Difference between revisions of "2016 AMC 10B Problems/Problem 19"
(→Problem) |
(→Problem) |
||
Line 26: | Line 26: | ||
\textbf{(D)}~\frac{10}{91}\qquad | \textbf{(D)}~\frac{10}{91}\qquad | ||
\textbf{(E)}~\frac19</math> | \textbf{(E)}~\frac19</math> | ||
+ | |||
+ | |||
+ | ==Solution== | ||
+ | <math>\textbf{(D)}~\frac{10}{91}</math> | ||
+ | |||
+ | solution by ngeorge |
Revision as of 09:29, 21 February 2016
Problem
Rectangle has and . Point lies on so that , point lies on so that . and point lies on so that . Segments and intersect at and , respectively. What is the value of ?
Solution
solution by ngeorge